2024 NAPLEX Practice Test - Answers
2024 NAPLEX Practice Test - Answers
https:// https://
www.amazon.com/ NAPLEX Crosswords NAPLEX Nuggets
www.amazon.
NAPLEX- Over 2,000 Engaging NAPLEX study book
Crosswords- NAPLEX Practice
com/NAPLEX- https://www.amazon.com/
NAPLEX-Nuggets-Meded101-
Engaging-Practice- Questions and Clues! Nuggets- Get on Amazon
Christianson-PharmD/dp/
B0BS9SX2JG/
Questions/dp/ https://www.amazon.com/NAPLEX-
Meded101-
B0CWYN1NL4/ Get on Amazon
Crosswords-Engaging-Practice-
Christianson-
Questions/dp/B0CWYN1NL4/
PharmD/dp/
B0BS9SX2JG/
Current lab work reveals a hemoglobin of 14.2 g/dL, creatinine of 1.1 mg/dL, B12 of 400 pg/mL, folic
acid of 15 ng/mL, and TSH of 26.5 mIU/L.
Current medications include: Synthroid 50 mcg daily, Zoloft (sertraline) 75 mg daily, and folic acid 1
mg daily.
The patient is taking all medications as directed. What is the most appropriate course of action?
Answer: C
Based on the presented symptoms and lab results, the patient is likely experiencing symptoms of
hypothyroidism, which is evidenced by the elevated TSH level.
The patient's B12 level is within the normal range, so there is no indication to add vitamin B12 at this
time, making Option B not recommended.
However, it is important for the patient to follow up with her healthcare provider for further
evaluation and monitoring.
2. How many mg of a medication will it take to create a 1,500 mL product at a concentration of 1:10,000?
Answer: 150 mg
For weight/volume (w/v) ratios, 1:100 (1%) means 1 gram per 100 mL.
Therefore, 1:10,000 (0.01%) means 1 gram per 10,000 mL
1 g/10,000 mL = 1,000 mg/10,000 mL = 0.1 mg/mL
Since 1,500 mL of total volume are needed at this concentration, multiply by 1,500 mL to determine
how many total milligrams are required.
A. Doxycycline
B. Bactrim
C. Erythromycin
D. Augmentin
Answer: B
Bactrim (sulfamethoxazole-trimethoprim) is the drug of choice for PCP prophylaxis.
4. Which of the following medications for diabetes would carry the least risk of hypoglycemia?
A. Glucophage
B. Glucotrol
C. Lantus
D. Humalog
Answer: A
Metformin is the least likely to cause hypoglycemia as it doesn’t stimulate insulin release. Glucotrol
(glipizide) is a sulfonylurea which directly stimulates beta cells to release insulin. Lantus and Humalog
are long acting and short acting insulin (respectively) so they certainly carry a risk of hypoglycemia.
5. A physician writes for a dose of Tylenol 20 grains twice daily. What dose of acetaminophen is this?
Answer: D
Remember the conversion of grains to milligrams is about 5 grains to 325 mg (1 grain = 64.8 mg).
A 48-year-old male has a history of bipolar disorder, Crohn’s disease, and was diagnosed with atrial
fibrillation within the last year. He was prescribed amiodarone for atrial fibrillation. Other medications
include Depakote, Lamictal, aspirin, and azathioprine.
6. Which medications could contribute to the lab abnormalities? Select all that apply.
A. Depakote
B. Aspirin
C. Azathioprine
D. Amiodarone
Answer: A, C and D
The elevated LFTs may be caused by amiodarone, Depakote (divalproex) or azathioprine. LFTs should
be monitored for these medications.
7. Based upon the medication list, which of the following is a significant interaction?
A. Depakote/Lamictal
B. Amiodarone/Aspirin
C. Azathioprine/Aspirin
D. Depakote/Azathioprine
Answer: A
Depakote (divalproex)/Lamictal (lamotrigine) is a major interaction. Depakote decreases Lamictal
clearance, and there is a recommended dosing regimen specific to starting Lamictal in a patient
already on Depakote.
A. Lamictal
B. Amiodarone
C. Depakote
D. Azathioprine
Answer: D
Azathioprine is an immunosuppressive medication and would be most likely to put this patient at risk
for infections.
9. In a patient with a creatinine clearance of less than 30 mL/min and ongoing edema that is not helped
by increasing the furosemide dose, which of the following would be the best option to help with
edema?
A. Adding hydrochlorothiazide
B. Adding Bumex
C. Adding metolazone
D. Adding captopril
Answer: C
Hydrochlorothiazide will likely not be effective given the poor kidney function. Bumex is a loop
diuretic and would be duplicate therapy as is already on furosemide. Captopril is an ACE inhibitor and
wouldn’t be of benefit for the edema.
Metolazone (Zaroxolyn) is the thiazide diuretic of choice to help with edema when Lasix resistance is
present.
10. Which of the following hypertension medications can also be beneficial for BPH symptoms?
A. Norvasc
B. Hytrin
C. Zestril
D. Lasix
Answer: B
Alpha blockers like Hytrin (terazosin) can be beneficial for both hypertension and BPH.
11. Which of the following is not a common physiological complication in the aging of an elderly patient?
Select all that apply.
A. Skin thickening, increasing the risk of skin and soft tissue infections
B. Slowed GI tract and possible reduced absorption of some vitamins and minerals like B12 and iron
C. A drop in GFR which can contribute to the accumulation of many medications
D. Increased concentration of albumin due to loss of intravascular fluid/dehydration
E. Loss of muscle mass, reduced physical conditioning
Answer: A and D
Skin thinning is more likely with aging, leading to possible risk of infections, abrasions, etc. Lower
blood albumin is more likely with malnourishment. This can increase the free fraction of many drugs
(warfarin, albumin).
A patient has been given a 500 mL bag that contains 60 mEq of potassium chloride. The IV is delivering 20
drops per mL at a rate of 8 drops per minute. It has been running for about 3 hours.
12. How many mEq of potassium has this patient been given?
13. How many mg of potassium chloride has this patient been given?
Answer: 644 mg
The molecular weight of potassium chloride is approximately 74.5 g/mol (39 g plus 35.5 g). To
convert the molecular weight of potassium chloride (KCl) into milliequivalents (mEq), one needs to
know the valence of each ion in the compound.
Potassium has a valence of +1, meaning that it can donate one positive charge (or one electron).
Chloride has a valence of -1, meaning that it can accept one negative charge (or one electron).
14. Which of the following medications would be likely to contribute to hyperkalemia, especially in
patients already receiving an ACE inhibitor or an ARB? Select all that apply.
A. Spironolactone
B. Trimethoprim
C. Lasix
D. Zaroxolyn
Answer: A and B
Trimethoprim can reduce the elimination of potassium. When used with an ACE inhibitor or ARB
(which are known to cause increased potassium levels) this effect can contribute to hyperkalemia.
Spironolactone is a potassium-sparing diuretic associated with an increased risk for hyperkalemia.
Lasix and Zaroxolyn are a loop and thiazide-like diuretic, respectively. They are more likely to cause
hypokalemia.
A. Case study, Case controlled series, Cohort study, Randomized controlled clinical trial
B. Randomized controlled clinical trial, Cohort study, Case controlled series, Case study
C. Cohort study, Randomized controlled clinical trial, Case study, Case controlled series
D. Case controlled series, Cohort study, Randomized controlled clinical trial, Case study
Answer: B
In drug study designs, double-blind, randomized controlled clinical trials provide the highest strength
of evidence due to their rigorous methodology, minimizing bias and confounding factors. Cohort
studies offer strong evidence but lack the randomization element. Case controlled series provide
moderate evidence as they are observational and retrospective. Single case studies offer the least
strength of evidence due to their anecdotal nature and lack of control groups.
16. An 85-year-old female is on metoprolol for atrial fibrillation, Bactrim for UTI prophylaxis, and
felodipine for hypertension. This patient had a recent cerebrovascular accident (CVA) and the primary
provider would like to start Coumadin on this patient.
A. 2.5 mg daily
B. 5 mg daily
C. 2.5 mg twice daily
D. 10 mg daily
Answer: A
General initial dosing of Coumadin (warfarin) is 2 to 5 mg daily. In the elderly, it is extremely
important to start Coumadin dosing low, and it is doubly important in this case as Bactrim can
interact (via CYP2C9 inhibition) and cause elevations in Coumadin concentrations in the body.
One could make the argument for an even lower starting dose, but given the options, 2.5 mg daily is
the most appropriate choice.
17. Which of the following medications is least likely to cause renal failure?
A. Augmentin
B. Gentamicin
C. Losartan
D. Celebrex
Answer: A
Augmentin (amoxicillin-clavulanate)is least likely to cause renal failure. Aminoglycosides, ARBs, ACEis,
and NSAIDs are all examples of medications that can contribute to renal failure.
A. B12 deficiency
B. Iron deficiency
C. Poor absorption in cachectic patients
D. Liver disease
Answer: B
The most common cause of erythropoietin failure in kidney disease is iron deficiency. Iron is an
essential nutrient for red blood cell production, and when the kidneys are not functioning properly,
they may not be able to regulate iron levels in the body. As a result, iron deficiency anemia can occur,
which can lead to erythropoietin resistance and failure.
While vitamin B12 deficiency and poor absorption can also contribute to anemia, they are less
common causes of erythropoietin failure in kidney disease. Liver disease is also not a common cause
of erythropoietin failure in kidney disease.
Iron stores should be assessed closely, especially in patients that are not seeing an increase in
hemoglobin.
19. A 55-year-old male with a history of alcoholism is diagnosed with liver failure. Current laboratory
testing reveals elevated ammonia levels.
What is the most appropriate therapy to treat the elevated ammonia level?
A. Propranolol
B. Metoclopramide
C. Lactulose
D. Thiamine
Answer: C
Propranolol is often used for treating portal hypertension in the management of esophageal varices,
but it will not lower ammonia levels. Thiamine is often supplemented in alcoholics, but it won’t lower
ammonia levels. Reglan (metoclopramide) is often used for treating GI conditions.
Of the available choices, lactulose is the only appropriate treatment for elevated ammonia.
A. Fentanyl
B. Oxycodone
C. Morphine
D. Methadone
Answer: C
Codeine is converted via CYP2D6 to morphine, which provides the analgesic effects.
Answer: C
Requip (ropinirole) is a dopamine agonist typically used for restless legs syndrome and occasionally
Parkinson’s disorder.
22. Which of the following side effects would be least likely with Aricept?
A. Nausea
B. Constipation
C. Weight loss
D. Cognitive changes
Answer: B
Aricept (donepezil) is an acetylcholinesterase inhibitor commonly used to treat dementia.
It is going to produce the opposite effects of anticholinergic medications. It would more likely cause
diarrhea, not constipation, as a side effect.
23. A 62-year-old Caucasian male presents to the hospital with NSTEMI. He became short of breath prior
to being admitted to the hospital. Biomarkers for troponin and creatine kinase (CK) were noted to be
elevated upon lab assessment.
Which of the following agents would be recommended upon discharge for this patient?
A. Atorvastatin
B. Aspirin
C. Metoprolol
D. Lisinopril
E. Isosorbide mononitrate
Answer: A, B, C and D
Statins, beta-blockers, antiplatelet agents (low-dose aspirin), and ACE inhibitors would all be
appropriate to initiate. Long-acting nitrates are not going to be part of a routine regimen post-MI.
A patient whose weight is 145 lbs and whose height is 5’3” is to receive a dose of Drug X.
The dosing for Drug X is 1200 mg/m2.
24. What dose (in mg) would be required for this patient? Round to the nearest hundred.
Answer: 2100 mg
The most commonly used equation to calculate body surface area (BSA) is the Mosteller formula,
which is:
25. What is this patient’s BMI? Round to the nearest tenth decimal place.
Answer: 25.7
Alternatively, weight can be measured in pounds and height can be measured in inches, but the
formula needs to be adjusted slightly to account for the difference in units:
BMI = (weight in pounds / (height in inches)²) x 703
Where 703 is a conversion factor that adjusts for the use of the imperial system (pounds and inches)
instead of the metric system (kilograms and meters).
A. Azithromycin
B. Ciprofloxacin
C. Doxycycline
D. Amoxicillin-clavulanate plus doxycycline
Answer: D
According to the 2019 IDSA/ATS Treatment Guidelines for Community-Acquired Pneumonia, the
choice of empiric therapy should be based on the severity of the patient's illness, risk factors for
drug-resistant pathogens, and local patterns of antimicrobial resistance.
The patient has a comorbid condition of type 2 diabetes, therefore monotherapy with azithromycin
or doxycycline would not be preferred. Additionally, given that the local resistance patterns to
macrolide therapy are not known, macrolide therapy would not be ideal.
Ciprofloxacin does not provide necessary coverage against S. pneumoniae, which is the predominant
pathogen for community acquired pneumonia.
27. A 55-year-old male with a past medical history of GERD, hypertension, and alcoholism is diagnosed
with a C. difficile infection. This is the first known episode.
A. Metronidazole
B. Oral vancomycin
C. Clindamycin
D. Doxycycline
Answer: B
According to the 2021 IDSA/SHEA Focused Update Guidelines on the Management of Clostridioides
difficile, fidaxomicin is the preferred treatment for an initial episode of C. difficile infection. When
fidaxomicin is not available or feasible, oral vancomycin is recommended as an alternative. Therefore,
in this case, since fidaxomicin is not an option listed, oral vancomycin is the most appropriate
therapy.
Metronidazole can be used for non-severe cases if the preferred treatments are unavailable or not
appropriate.
Note: It is commonly thought that consumption of alcohol while taking metronidazole or tinidazole
could result in a highly unpleasant disulfiram-like reaction; however, this warning, which is included
in the FDA prescribing information, was based on older laboratory experiments and individual case
histories where the reactions were equally likely to be due to alcohol alone. Current
recommendations from the CDC suggest that avoiding alcohol is unnecessary, as review of available
data suggests there is no convincing evidence of a disulfiram-like reaction between metronidazole or
tinidazole and alcohol. Further, neither metronidazole or tinidazole inhibit acetaldehyde
dehydrogenase (unlike disulfiram), making them incompatible with producing the disulfiram-like
reaction.
© Med Ed 101, Inc.
28. A 74-year-old female is currently taking the following medications: famotidine, diltiazem,
amiodarone, and Tylenol. She has been experiencing edema of late.
A. Famotidine
B. Diltiazem
C. Amiodarone
D. Tylenol
Answer: B
Calcium channel blockers (CCBs) are a notorious cause of edema. Also, it is important to note that
diltiazem (non-dihydropyridine CCBs) can be used in atrial fibrillation.
29. A 32-year-old female presents to the pharmacy with a bottle of St. John’s wort. She is currently on
birth control containing estrogen.
A. St. John’s wort is an enzyme inhibitor and can reduce estrogen concentrations in the body
B. St. John’s wort is an enzyme inducer and can reduce estrogen concentrations in the body
C. St. John’s wort is an enzyme inhibitor and can increase estrogen concentrations in the body
leading to increased risk of adverse effects
D. St. John’s wort is an enzyme inducer and can increase estrogen concentrations in the body leading
to increased risk of adverse effects
Answer: B
St. John's wort is a natural remedy available over-the-counter used to improve mood and lower
anxiety. However, it is known to interact with many medications, including birth control pills that
contain estrogen.
St. John's wort induces the activity of enzymes in the liver that metabolize estrogen and other
hormones. This can lead to decreased effectiveness of birth control pills, as the rapid metabolism of
estrogen can lead to lower blood levels of the hormone, potentially reducing the effectiveness of the
contraceptive.
30. Which of the following medication patches would it be appropriate to cut and still use?
A. Fentanyl
B. Exelon
C. Lidoderm
D. Scopolamine
Answer: C
Lidoderm (lidocaine patch) is the only one that would be appropriate to cut. Lidoderm patches may
be cut into smaller sizes with scissors prior to removal of the release liner.
Cutting into the other patches (which are extended-release formulations) may lead to higher rates of
drug release, increasing the risk for serious side effects (and overdose, in the case of fentanyl).
A. Zoloft
B. Lexapro
C. Wellbutrin
D. Pamelor
Answer: C
Clinical studies have shown that bupropion has a lower risk of causing sexual dysfunction compared
to other antidepressants. In fact, some studies have shown that bupropion may even improve sexual
function in some patients.
32. Which of the following medications can increase the risk for osteoporosis?
A. Zofran
B. Prednisone
C. Tramadol
D. Methotrexate
Answer: B
Long-term use of corticosteroids is known to increase the risk for developing osteoporosis.
33. The prescription for eye drops calls for 2 drops in the left eye twice daily. Given the following choices
of package size, how many milliliters should be dispensed to get at least a month’s supply while
minimizing excess?
A. 2.5 mL
B. 5 mL
C. 7.5 mL
D. 10 mL
Answer: C
In this case, 4 drops are needed per day.
Typically, each drop is about 0.05 mL (20 drops per mL). 6 mL would provide about 120 drops. 5 mL
would not provide enough drops for the full month, so 7.5 mL would be the most appropriate
package size.
34. A 33-year-old male with a history of asthma was recently started on an albuterol inhaler. Which of the
following would be an important education point?
A. Rinse mouth with water and and spit following administration to prevent thrush
B. This medication may interact with over the counter antitussives, like Robitussin
C. This medication may cause an short-term increase in heart rate
D. This medication should not be used in patients with a history GERD
Answer: C
Rinsing the mouth with water and spitting following each dose is intended to prevent thrush from
inhaled corticosteroids (e.g., budesonide, fluticasone, etc.), not inhaled albuterol. A history of GERD
and/or use of OTC cough medications is not expected to interact with this patient’s albuterol.
A. Carafate
B. Sevelamer
C. Calcitriol
D. Miacalcin
Answer: B
Sevelamer is a phosphate binder used to help treat high phosphorus levels in the body (usually
caused by chronic kidney disease). Calcitriol is a vitamin D analog usually used for
hyperparathyroidism or hypocalcemia issues. Carafate is used to help alleviate GI symptoms.
Miacalcin is used in the treatment of osteoporosis.
36. Which of the following would be appropriate monitoring for adverse effects of hydroxychloroquine?
Answer: B
Hydroxychloroquine is an antimalarial drug that is also used to treat autoimmune conditions such as
rheumatoid arthritis and lupus. One of the potential adverse effects of hydroxychloroquine is
retinopathy, a condition that affects the retina of the eye and can lead to vision loss.
Therefore, it is recommended to have regular eye exams before and during treatment with
hydroxychloroquine to monitor for any signs of retinopathy.
Routine foot exams, CPK monitoring, and neuropathy monitoring are not typically indicated for
adverse effects of hydroxychloroquine.
37. Which of the following would be least appropriate for the management of acute pain?
A. Ibuprofen
B. Duragesic
C. Oxycodone
D. Tramadol
Answer: B
Duragesic (fentanyl transdermal) is only indicated for management of pain in opioid-tolerant patients
that require daily, around-the-clock, long-term opioid treatment for which alternative treatment
options are inadequate. Additionally, it has a very slow onset that would take hours to a day or two
for a patient to start feeling the analgesic effects. For these reasons, it would not be appropriate for
treating a patient who is in acute pain.
A. Duloxetine
B. Amitriptyline
C. Pregabalin
D. Neurontin
Answer: B
While TCAs like amitriptyline might be useful for neuropathy in certain patients, they are highly
anticholinergic and could lead to urinary retention which is not ideal in a patient with BPH.
Amitriptyline is one of the most highly anticholinergic TCAs.
A. Celebrex
B. Ketorolac
C. Acetaminophen
D. Ultram
Answer: B
Celebrex (COX-2 inhibitor) tends to be a little less irritating on the GI tract than traditional NSAIDs.
Ketorolac is one of the worst NSAIDs in terms of GI irritation/bleeds. Tylenol is one of the safest
analgesics in the elderly. Tramadol will likely not cause a GI bleed as well.
40. Capsaicin is a topical product used in the treatment of muscle pain. What is its primary mechanism of
action?
Answer: B
Capsaicin is a natural compound found in hot peppers, and it is believed to work by depleting
substance P, a neurotransmitter that transmits pain signals to the brain. Depletion of substance P
takes a significant amount of time. This medication will likely not be effective for acute pain relief or
when used on an as-needed basis.
41. A 77-year-old male with a history of multiple DVTs is currently receiving warfarin therapy. He is
diagnosed with osteomyelitis and treatment with rifampin is initiated.
A. INR will go up
B. INR will go down
C. INR will be unaffected
D. Too little information to tell
Answer: A
Rifampin is a classic enzyme inducer. When it is initiated, it will likely cause Coumadin/INRs to go
down. When it is discontinued, it will stop inducing metabolism and INRs will likely increase.
© Med Ed 101, Inc.
42. A 57-year-old male has been initiated on Aldactone. What should be monitored?
A. Liver function
B. Potassium levels
C. Albumin
D. Pulses
Answer: B
Aldactone (spironolactone) is a potassium-sparing diuretic and can cause hyperkalemia. Check
potassium levels within 1 week of initiation or titration and regularly thereafter. More frequent
monitoring may be needed when given with other drugs that cause hyperkalemia or in patients with
impaired renal function.
Answer: B
HIPAA was designed to prevent private healthcare information from being shared in a public forum or
for unauthorized purposes.
44. A patient that is a poor CYP3A4 metabolizer would have what effect on the use of Plavix?
Answer: C
Plavix (clopidogrel) is a prodrug metabolized by CYP2C19. In this case, it wouldn’t matter if the
patient was a poor CYP3A4. If the patient was a poor CYP2C19 metabolizer, it would likely lead to
reduced medication efficacy.
45. What class of medication would likely be used for esophageal varices associated with liver cirrhosis?
A. ACE inhibitor
B. Alpha blocker
C. Calcium channel blocker
D. Beta-blocker
Answer: D
Specifically, non-selective beta blockers like nadolol and propranolol would be used for esophageal
varices prophylaxis.
A. Amiodarone
B. Hydralazine
C. Nabumetone
D. Imodium
Answer: A
Amiodarone can cause pulmonary fibrosis (sometimes called blue man syndrome).
47. A 66-year-old female was recently started on Lamictal. What is an important educational point that
this patient should look out for?
Answer: B
Lamotrigine can cause severe skin rashes that require hospitalization and discontinuation of
treatment. Almost all cases of life-threatening rashes caused by lamotrigine occurred within 2 to 8
weeks after starting treatment. However, isolated cases appeared after long-term treatment.
Because it is not possible to predict which rashes will prove to be serious or life threatening, advise
patients to contact their provider immediately if any sign of rash occurs.
A. Methylene blue
B. Phytonadione
C. Protein C
D. Protamine sulfate
Answer: D
Protamine sulfate works by binding to and neutralizing the effects of heparin, which helps to reverse
the anticoagulant effects of heparin.
49. Which of the following medications works by antagonizing muscarinic receptors in the bladder?
A. Ditropan
B. Phenazopyridine
C. Myrbetriq
D. Flomax
Answer: A
Ditropan (oxybutynin) is an anticholinergic or antimuscarinic medication used in the treatment of
overactive bladder. Flomax (tamsulosin) is an alpha blocker, Myrbetriq (mirabegron) is a beta agonist,
and phenazopyridine is a urinary analgesic.
A 78-year-old white male with a history of atrial fibrillation, hypertension, GERD, and CHF is currently
receiving metoprolol 25 mg twice daily, digoxin 0.25 mg daily, omeprazole 20 mg daily, Pepcid 40 mg at
bedtime, and aspirin 81 mg daily.
Recent labs reveal an elevation in creatinine of 1.8 mg/dL. 6 months ago, creatinine was 1.1 mg/dL.
Other information:
● Height = 6 feet 2 inches
● Weight 178 lbs
● Blood pressure = 128/78 mmHg
● Pulse = 62 bpm
● Respiration rate = 21 breaths per minute
50. Which medication is this change in creatinine most likely to require dose adjustments?
A. Metoprolol
B. Pepcid
C. Omeprazole
D. Aspirin
Answer: B
Pepcid (famotidine) has renal dose adjustments for patients with eCrCl ≤ 60 mL/min (in this example,
the eCrCl is ~39 mL/min). The other medications listed are not renally dosed. Aspirin could be a
concern, if used at analgesic doses, but this is not likely at a dose of 81 mg daily.
51. Which of the following is not consistent with signs of digoxin toxicity?
A. Nausea
B. Vertical nystagmus
C. Bradycardia
D. Weight loss
Answer: B
Vertical nystagmus is a unique sign of Dilantin (phenytoin) toxicity, not digoxin. Weight loss,
confusion, bradycardia, and GI effects are all consistent with signs of digoxin toxicity.
Digoxin is cleared by the kidney so digoxin levels should be monitored closely given the change in
kidney function.
52. Rounding to the nearest whole number, what is the estimated creatinine clearance in this patient
(mL/min)?
Answer: 39 mL/min
A. Lantus
B. Pioglitazone
C. Glucotrol
D. Victoza
Answer: B
The TZDs like pioglitazone (Actos) should be avoided if possible in patients with congestive heart
failure.
54. A 7-year-old male weighing 24 kg is experiencing a significant fever of 103 degrees Fahrenheit. What
is the most appropriate dose of Tylenol to initiate?
A. 100 mg
B. 300 mg
C. 500 mg
D. 700 mg
Answer: B
The pediatric dosing of acetaminophen is 10-15 mg/kg/dose. In the above case, it would be 24 kg x
10–15 mg/kg/dose giving a range of 240 to 360 mg.
55. A new medication for Parkinson’s disease is being investigated. Researchers are trying to identify the
half-life of the drug. The elimination rate constant is 0.32 per hour. What is the approximate half-life?
Answer: D
Half-life = 0.693/k (where k is the elimination rate constant)
Half-life = 0.693 / 0.32 per hour = 2.17 hours = ~2 hours
56. Approximately how many grams of NaCl would need to be added to 330 mL of pure water to create
an isotonic solution?
A. 3 grams
B. 4 grams
C. 5 grams
D. 6 grams
Answer: A
Isotonic solution of NaCl has 0.9 grams per 100mL (also called normal saline or NS 0.9%). To calculate
the amount of NaCl needed to achieve the same concentration in 330 mL of solution,
cross-multiplication can be used.
Which of the following supplements would be the highest risk for her to try?
A. Glucosamine
B. Garlic
C. Thiamine
D. Vitamin C
Answer: B
Being on both Coumadin (warfarin) and aspirin this patient is at a high risk of bleeding. Garlic,
ginseng, and gingko (3G’s) are supplements that can increase the risk of bleeding. It would be most
advisable to avoid garlic use in this case.
58. A 19-year-old female presents about 6 hours after ingesting a large, unknown amount of
acetaminophen. She is extremely nauseated.
Answer: D
Coal tar can bind up drugs in overdose situations but needs to be done very quickly (usually within
about an hour of ingestion). Ipecac use is not recommended.
Checking an acetaminophen level is useful to monitor, but given that there is a known ingestion and
symptoms of toxicity administering the antidote (N-acetylcysteine) would be the best initial course of
action.
59. Which of the following is not true regarding the use of long acting nitrates?
Answer: D
Concomitant use with PDE-5 inhibitors (such as Viagra and Cialis) can lead to severe hypotension and
an increased fall risk.
A 45-year-old female with a history of epilepsy, hypertension, and diabetes is taking the following
medications. Dilantin 200 mg twice daily, lisinopril 10 mg daily, metformin 500 mg twice daily, and
glipizide 5 mg daily. She was prescribed fluconazole 100 mg twice daily for 14 days about 5 days ago. She
has been feeling weak, nauseated, slurring speech, and having difficulty with her gait.
Answer: C
Diflucan (fluconazole) is a moderate CYP2C9 and CYP3A4 inhibitor and a common cause of drug
interactions. It can actually cause elevated drug levels of both glipizide and Dilantin (phenytoin).
Hypoglycemia could be a possibility, but with a blood sugar of 99 mg/dL, that is unlikely.
The total Dilantin level is elevated (normal range 10-20 mcg/mL) and the patient is displaying
symptoms of toxicity.
61. In the patient above, chronic Dilantin use can contribute to which deficiency?
A. Folate
B. B12
C. Vitamin C
D. Vitamin D
Answer: D
Phenytoin induces hepatic metabolizing enzymes. This may enhance the metabolism of vitamin D
and decrease vitamin D levels. For these reasons, the chronic use of phenytoin has been associated
with decreased bone mineral density (osteopenia, osteoporosis, and osteomalacia) and bone
fractures.
Answer: A
Congestive heart failure is a condition in which the heart is not able to pump blood effectively,
leading to fluid accumulation in the lungs and other parts of the body.
A. 12 kcals
B. 27 kcals
C. 120 kcals
D. 270 kcals
Answer: D
● Lipids = 9 kcals/gram
● Dextrose = 3.4 kcal/gram
● Protein = 4 kcal/gram
64. A patient with confirmed HIV infection is starting antiretroviral therapy. Their HLA-B*5701 test is
positive. Which of the following regimens would not be appropriate?
A. Biktarvy
B. Triumeq
C. Dovato
D. Dovato plus Viread
Answer: B
HLA-B*5701 testing is a crucial genetic screening conducted before initiating abacavir-containing
antiretroviral therapy (ART) regimens. This test is important because individuals who have the
HLA-B*5701 allele are at a significantly increased risk of developing a hypersensitivity reaction to
abacavir, which is potentially life-threatening.
By screening for the HLA-B*5701 allele before starting abacavir, those at high risk for hypersensitivity
can be identified and alternative medications can be chosen, significantly reducing the risk of this
serious reaction.
A. Bumetanide
B. Triamterene
C. Metolazone
D. Ethacrynic acid
Answer: C
Bumex (bumetanide) and ethacrynic acid are loop diuretics. Triamterene is a potassium-sparing
diuretic. Metolazone is a thiazide-like diuretic.
66. What is the half-life of a medication if the concentration at 8 AM is 16 mg/L and 12 hours later at 8
PM the concentration is at 2 mg/L?
A. 1 hour
B. 2 hours
C. 3 hours
D. 4 hours
Answer: D
The half-life of a medication is the time required for the concentration or amount to decrease by half
its initial value.
The initial concentration is 16 mg/L. One half-life later, the concentration would be 8 mg/L. The
concentrations at the second and third half-life would be 4 mg/L and 2 mg/L. This means that it takes
3 half-lives (and 12 hours) to go from 16 mg/L to 2 mg/L.
A. Person who has had vaginal or anal sex (with inconsistent condom usage) in the last 6 months with
a partner of unknown HIV status
B. Person who uses injectable drugs (illicit substances) and shares injection equipment
C. Person who has been diagnosed with gonorrhea or syphilis in the past 6 months
D. All the above
Answer: D
According to the US Public Health Service, all of the persons described above could be considered for
PrEP therapy to prevent HIV infection.
68. Which of the following is a common complication with the use of paclitaxel?
A. Retinopathy
B. Neuropathy
C. Rash
D. Cystitis
Answer: B
Paclitaxel is a chemotherapy drug that can cause damage to the peripheral nerves, leading to
neuropathy, a condition that is characterized by symptoms such as tingling, numbness, pain, and
weakness.
© Med Ed 101, Inc.
69. A primary provider is asking you for your opinion on which would be the most appropriate
antihypertensive in a patient who has frequent gout flares. Which would be least appropriate?
A. Metoprolol
B. Lisinopril
C. Chlorthalidone
D. Clonidine
Answer: C
Thiazide diuretics can elevate uric acid and potentially contribute to gout flares. For this reason,
chlorthalidone should be avoided in patients that have uncontrolled gout/frequent flares (if
possible).
A 45-year-old male presents to the clinic with an inflamed and extremely painful left big toe.
70. What medication would be the most appropriate to use to treat this patient’s pain?
A. Allopurinol
B. Indomethacin
C. Tylenol
D. Methotrexate
Answer: B
Based on patient presentation (extreme pain located in the big toe plus elevated uric acid levels) this
is likely an acute gout flare. Allopurinol is used for uric acid-lowering therapy and it will not help with
the acute pain in this situation. NSAIDs (such as indomethacin) are used to treat acute gout pain.
Steroids or colchicine are other options in the event of NSAID contraindication.
71. Which of the following medications should not be used in a patient who has a stated sulfa allergy?
A. Unasyn
B. Levaquin
C. Septra
D. Doxycycline
Answer: C
Unasyn is a combo of ampicillin/sulbactam.
Septra is sulfamethoxazole and trimethoprim (also known as Bactrim) and should not be used in a
patient with a sulfa allergy.
A. Inhibits Factor Xa
B. Bind to ADP receptors on platelets
C. Direct thrombin inhibitor
D. None of the above
Answer: A
Xarelto produces its anticoagulant effect by inhibiting factor Xa.
73. Which of the following medications would most likely increase the risk of deep vein thrombosis (DVT)?
A. Lopressor
B. Premarin
C. Diclofenac
D. Probenecid
Answer: B
Premarin is an estrogen product and poses the greatest likelihood for increasing the risk of DVT.
A. CYP3A4 inhibition
B. P-glycoprotein inhibition
C. Blocking absorption in the gut
D. Reducing elimination via the kidney
Answer: C
Questran (cholestyramine) is a lipid lowering agent that binds to bile acids (metabolites of
cholesterol) in the intestinal lumen and prevents their reabsorption. Due to the mechanism of action,
patients should take other drugs at least 1 hour before or 4 hours after bile acid sequestrants to
avoid impeding their absorption.
75. Which of the following statins can be considered high-intensity lipid lowering agents?
Select all that apply.
A. Atorvastatin
B. Simvastatin
C. Pravastatin
D. Rosuvastatin
Answer: A and D
Atorvastatin (40-80 mg) and rosuvastatin (20-40 mg) are the only options that can be considered
high-intensity lipid lowering agents.
A. Aranesp
B. Erythropoietin
C. Remicade
D. Neupogen
Answer: D
Neupogen (filgrastim) is a colony stimulating factor (CSF) used to help treat neutropenia. Aranesp
(darbepoetin) and erythropoietin are used to treat anemia and stimulate the production of red blood
cells. Remicade (infliximab) is an immunosuppressant used for treating inflammatory conditions (e.g.,
arthritis, ulcerative colitis).
77. According to the DEA, which medication has the lowest potential for abuse?
A. Ativan
B. Methadone
C. Ritalin
D. Kadian
Answer: A
Kadian (extended-release morphine), Ritalin (methylphenidate) and methadone all are schedule 2
controlled substances. Ativan (lorazepam) is a schedule 4 controlled substance and would therefore
have the lowest potential for abuse.
A. Anxiety
B. Nausea
C. Constipation
D. Sweating
Answer: C
Constipation is not likely to be a symptom of opioid withdrawal (diarrhea is more likely).
A 67-year-old African American male has a history of hypertension and bipolar disorder. He is currently
taking hydralazine, hydrochlorothiazide, loratadine and lithium. He has been feeling a little off lately.
Labs/vitals reveal:
● Creatinine = 1.1 mg/dL
● Potassium = 4.0 mEq/L
● Sodium = 142 mEq/L
● TSH = 17.4 mIU/mL
● Lithium 0.9 mEq/L
● BP = 156/78 mmHg
A. Hydralazine
B. Lithium
C. Hydrochlorothiazide
D. Loratadine
Answer: B
Lithium is concentrated within the thyroid and can inhibit thyroid synthesis and release which can
lead to hypothyroidism. Based on the elevated TSH level and normal lithium level the patient is most
likely experiencing symptoms of hypothyroidism.
Thyroid function should be assessed before the initiation of lithium treatment, at three months and
every six to twelve months while treatment is ongoing. If serum thyroid tests warrant concern,
monitoring should occur more frequently.
From the labs, this patient is likely experiencing changes in thyroid function due to the lithium. TSH
should be monitored with lithium (amiodarone is another classic drug that can affect thyroid
function).
80. Of the other medications, which medication would be most likely to cause lithium levels to rise?
A. Hydralazine
B. Loratadine
C. Hydrochlorothiazide
D. None of the above would interact
Answer: C
Diuretics increase the excretion of sodium in the urine, leading to a decrease in blood sodium
concentration. This can cause an increase in the reabsorption of sodium and lithium in the kidneys
since the body tries to conserve these electrolytes.
Diuretics can also cause a decrease in extracellular fluid volume, triggering the release of hormones
that regulate the reabsorption of sodium and water in the kidneys, which can also affect the
reabsorption of lithium. As a result, the net effect of diuretics on serum lithium concentrations is an
increase in lithium reabsorption in the kidneys, which can lead to an accumulation of lithium in the
body and an increase in serum lithium concentrations.
Close monitoring of serum lithium levels is necessary in patients taking both diuretics and lithium,
and medication regimens may need to be adjusted accordingly.
A. Lisinopril
B. Amlodipine
C. Losartan
D. Aliskiren
Answer: B
Given that the patient is an African American male and is already receiving a thiazide diuretic,
replacing hydralazine with a calcium channel blocker (such as amlodipine) is the most appropriate
option.
Additionally, addition of an ACE inhibitor or ARB in a patient on lithium could cause elevated lithium
levels.
82. Which of the following statins is least likely to be involved in a drug interaction?
A. Atorvastatin
B. Simvastatin
C. Pravastatin
D. Rosuvastatin
Answer: C
Pravastatin is the only agent listed that is not metabolized by CYP enzymes. Compared to the other
statins, it is least likely to be involved in a drug interaction.
Answer: A
Often referred to as Kayexalate, sodium polystyrene sulfonate is used in the treatment of
hyperkalemia.
84. A 25-year-old patient presents to the pharmacy stating she has a head cold and that she has been
using Afrin for about 2 days since the symptoms started. She is not experiencing any benefit for her
nasal congestion. She has no fever and she rates the pain at 2 on a scale of 1-10.
Rebound congestion occurs with nasal spray decongestants (after 3-5 days of use) and not with oral
nasal decongestants because of the way that nasal spray decongestants work and are absorbed in the
body.
Nasal spray decongestants work by constricting the blood vessels in the nasal tissues, which reduces
the inflammation and swelling that causes congestion. However, with prolonged use, the blood
vessels can become dependent on the medication to maintain their constriction, and when the
medication is discontinued, the blood vessels rebound and dilate, leading to worsening congestion.
In contrast, oral nasal decongestants, such as pseudoephedrine, work by constricting the blood
vessels throughout the body, including those in the nasal tissues. Since these medications are
absorbed systemically, they do not have the same localized effect on the nasal tissues as nasal spray
decongestants. Therefore, they are less likely to cause rebound congestion.
85. Which of the following is least likely to alleviate acute respiratory symptoms in COPD?
A. Albuterol
B. Xopenex
C. Flovent
D. Ipratropium
Answer: C
Flovent is an inhaled corticosteroid. Remember that inhaled corticosteroids are controller type
medications that reduce inflammation and will not work quickly to alleviate acute respiratory
distress.
Answer: A
The recommended administration for Lidoderm patches is 12 hours on, 12 hours off. Leaving the
patch on for 24 hours is not recommended.
87. A 2-year-old male patient is diagnosed with acute otitis media. What is the recommended first line
treatment?
A. Levofloxacin
B. Amoxicillin
C. Bactrim
D. Keflex
Answer: B
Amoxicillin is effective against the most common bacterial pathogens that cause acute otitis media. It
is generally well-tolerated and has a low incidence of side effects in children. Levofloxacin, Bactrim,
and Keflex are not recommended as first-line treatments for acute otitis media in children due to
their potential side effects and lower efficacy against the most common bacterial pathogens causing
this condition.
© Med Ed 101, Inc.
88. A 33-year-old female has a history of hypertension, GERD, and depression. Current medications
include: omeprazole, lisinopril, TUMS (as-needed), and Zoloft. She just found out that she is
pregnant.
A. Lisinopril
B. Omeprazole
C. Tums
D. Zoloft
Answer: A
Lisinopril is contraindicated in pregnancy due to the risk for fetal abnormalities and should be
discontinued immediately. The other medications are generally considered safe during pregnancy
(under medical supervision) but all medication use should still be assessed and discussed.
89. Which of the following medications works by selectively inhibiting serotonin reuptake inhibitors in the
treatment of depression?
A. Effexor
B. Cymbalta
C. Luvox
D. Elavil
Answer: C
Luvox (fluvoxamine) is the only SSRI listed. Effexor (venlafaxine) and Cymbalta (duloxetine) are SNRIs
and Elavil (amitriptyline) is a TCA.
Medications include:
● Amlodipine
● Lisinopril
● Ibuprofen
● Gabapentin
Which medication would least likely be the cause of the peripheral edema?
A. Amlodipine
B. Lisinopril
C. Ibuprofen
D. Gabapentin
Answer: D
Of the listed medications, lisinopril is least likely to be the cause of peripheral edema.
An exhausted mother is with her 8 month old child. She has been crying and just got back from the
physician’s office in which they told her to use OTC pain meds as she is just teething. The 8 month old has
no known allergies and is 16 lbs.
Answer: B
Of the available options, 50 mg every 6 hours as needed is the most appropriate dosing.
92. With her stuffy nose, she is having an occasional cough. Her mother would like an option to help with
this. What is the most appropriate recommendation?
A. Pseudoephedrine
B. Diphenhydramine
C. Dextromethorphan
D. Cough Drops
E. None of the above
Answer: E
The cough does not appear to be very significant. Treating mild to moderate cough in infants with
over the counter systemic agents is not recommended. Cough drops would not be appropriate for an
8 month old.
93. A 55-year-old male is diagnosed with Lyme’s disease. What is the most appropriate initial treatment?
A. Valtrex
B. Vancomycin
C. Doxycycline
D. Clindamycin
Answer: C
Doxycycline is typically the first line treatment for Lyme’s disease.
94. Which bacteria is associated with the highest incidence of causing UTIs?
A. S. pneumoniae
B. S. aureus
C. P. aeruginosa
D. E. coli
Answer: D
E. coli is the most common cause of UTIs.
© Med Ed 101, Inc.
95. A 44-year-old male is on the following medications:
● Hydrochlorothiazide
● Tramadol
● Quetiapine
● Prozac
He has had a weight gain of 20 pounds in the last 6 months. Which medication is the most likely cause?
A. Hydrochlorothiazide
B. Tramadol
C. Quetiapine
D. Prozac
Answer: C
Antipsychotics as a class can contribute to metabolic syndrome, weight gain and increase the risk for
developing diabetes. Antidepressants, such as SSRIs, can cause weight gain, but generally less so than
antipsychotics. Quetiapine is the most likely cause.
96. Which of the following immunizations is recommended annually for patients greater than 65?
A. Pneumococcal
B. Influenza
C. Varicella
D. Prevnar
Answer: B
Influenza is the only one that is recommended annually.
97. A fast metabolizer via CYP3A4 would expect what effect on concentrations of erythromycin versus the
normal population?
A. Higher
B. Lower
C. No change
D. None of the above
Answer: B
Erythromycin is primarily broken down via CYP3A4. A fast metabolizer of CYP3A4 will break down the
drug faster, leading to lower concentrations compared to the general population.
Answer: D
Famotidine and the others are H2 receptor antagonists that work primarily in the stomach to reduce
gastric acid secretion. H1 receptor antagonism is the mechanism of action of antihistamines, such as
Benadryl and Claritin, which are used for managing allergy symptoms.
A. Hyperglycemia
B. GI upset
C. Liver toxicity
D. Insomnia
Answer: C
Liver toxicity would not be common with systemic corticosteroids. The others are very important side
effects to remember when educating patients.
A. Varicella
B. FluMist
C. Tdap
D. MMR
Answer: C
Tdap is the only one of the four that is not a live vaccine. It is important to remember given that
immunocompromised patients should generally not receive live vaccines.
A 61-year-old female has a history of osteoporosis, GI bleeding, knee osteoarthritis, gout, and
hypertension. She presents to the pharmacy with worsening pain in her left knee. She hasn’t taken
anything to treat her pain at this point because she is scared that anything over the counter she takes will
interact with her medications.
Her current medications include Fosamax, calcium with vitamin D supplement, losartan, metoprolol,
allopurinol, prednisone, and hydrochlorothiazide.
101. What would be the most appropriate and effective long-term treatment for her osteoarthritis?
A. Ketorolac
B. Tramadol
C. Topical diclofenac
D. Acetaminophen
Answer: C
In this patient, it is preferable to use a topical product when pain is localized to help minimize the risk
for systemic adverse effects. Ketorolac is an NSAID with a high risk for GI bleeding, which is not ideal
given the patient’s medical history. Additionally, it can only be used for up to 5 days consecutively,
which makes it a poor option when chronic use is necessary. Tramadol may be considered
down-the-line, but is generally not a first-line treatment option. Acetaminophen has fallen out of
favor for use in osteoarthritis due to a lack of efficacy.
A. Prednisone
B. Hydrochlorothiazide
C. Losartan
D. Fosamax
Answer: B
Hydrochlorothiazide has the greatest potential to elevate uric acid levels among the medications
listed. Thiazide diuretics, such as hydrochlorothiazide, increase uric acid levels by reducing the
excretion of uric acid by the kidneys. Both ACE inhibitors and ARBs (such as losartan) can affect the
renal system, which may lead to decreased excretion of uric acid and subsequent increases in serum
uric acid levels.
However, the effect of these medications on uric acid levels is generally considered to be small.
Fosamax (alendronate) is not expected to affect uric acid levels.
103. Out of the following medications, which is most likely to cause increases in calcium levels?
A. Hydrochlorothiazide
B. Losartan
C. Prednisone
D. Tylenol
Answer: A
Thiazide diuretics (such as hydrochlorothiazide) can cause an increase in calcium levels by increasing
renal tubular reabsorption of calcium. This is different from loop diuretics, which can lead to
decreased calcium levels by enhancing renal excretion.
Answer: D
Bisphosphonates (such as alendronate) inhibit osteoclast activity. Remember that osteoclast activity
essentially leads to the breakdown of bones and osteoblasts “build” bone.
A. Zoloft
B. Elavil
C. Duloxetine
D. Trazodone
Answer: C
Of the medications listed, duloxetine would be the most appropriate for a patient reporting
symptoms of depression and neuropathy. Duloxetine is a serotonin-norepinephrine reuptake
inhibitor (SNRI) that is approved for the treatment of depression and several types of neuropathic
pain, including diabetic peripheral neuropathy.
Elavil (amitriptyline) is a tricyclic antidepressant (TCA) that is sometimes used for the treatment of
neuropathic pain, but it has a higher risk of side effects compared to other medications, particularly
in older adults.
Zoloft (sertraline) is a selective serotonin reuptake inhibitor (SSRI) that is primarily used for the
treatment of depression and anxiety disorders, but it is not typically used for neuropathic pain.
Trazodone is another antidepressant that is sometimes used for the treatment of depression and
insomnia, but it is not typically used for neuropathic pain.
106. Which of the following is the most common side effect of interferon beta-1a?
A. Loose stools
B. Flu-like symptoms
C. Leukopenia
D. Dry mouth
Answer: B
Interferon beta-1a is a medication used to treat multiple sclerosis, and it works by reducing
inflammation in the central nervous system. However, it can also cause a range of side effects, the
most common of which are flu-like symptoms such as fever, chills, headache, muscle aches, and
fatigue. These symptoms can be managed with acetaminophen or nonsteroidal anti-inflammatory
drugs (NSAIDs) taken prior to the injection and continued as needed for several days afterwards.
Which of the following agents would be most appropriate to initiate to safely lower her blood
pressure <185/110 mmHg?
A. Nitroglycerin
B. Labetalol
C. Hydralazine
D. Nitroprusside
Answer: B
The 2019 Update to the AHA/ASA Acute Ischemic Stroke Guidelines lists labetalol, nicardipine,
clevidipine, hydralazine and enalapril as therapeutic options to lower blood pressure in patients
eligible for thrombolytic therapy. Of the listed agents, labetalol is the most appropriate selection.
Labetalol is preferred over hydralazine for lowering blood pressure in patients with acute ischemic
stroke because it has a more predictable and stable blood pressure-lowering effect. Labetalol is a
nonselective beta-blocker with alpha-blocking properties, which makes it effective in lowering blood
pressure without causing reflex tachycardia. On the other hand, hydralazine is a direct-acting
vasodilator that can cause reflex tachycardia and increase myocardial oxygen demand, which can be
harmful in patients with acute ischemic stroke. Additionally, labetalol has been shown to be more
effective than hydralazine in achieving rapid blood pressure control in patients with acute ischemic
stroke. Therefore, labetalol is the preferred agent for blood pressure control in patients with acute
ischemic stroke, especially those who are candidates for thrombolytic therapy.
108. A 21-year-old male presents to the emergency department with incredibly painful head and neck
pain. Upon assessment by the primary provider, it is determined that he likely has bacterial
meningitis.
Which of the following would be the agent of choice to initiate (in addition to vancomycin)?
A. Clindamycin
B. Levofloxacin
C. Doxycycline
D. Ceftriaxone
Answer: D
For this patient, the use of vancomycin plus a third-generation cephalosporin (such as ceftriaxone) is
the most appropriate empiric therapy for bacterial meningitis.
A. Prozac
B. Famotidine
C. Vitamin B6
D. Minocycline
Answer: D
Minocycline should be stopped immediately in this scenario.
Minocycline is an antibiotic in the tetracycline class, which can cross the placenta and potentially
harm the developing fetus. It has been associated with tooth discoloration, inhibition of bone
growth, and other adverse effects in fetuses exposed to the medication during pregnancy.
In this scenario, the patient is using minocycline as a treatment for acne. There are topical acne
treatments safe for use during pregnancy that can replace the minocycline (such as topical benzoyl
peroxide).
Prozac (fluoxetine) is a selective serotonin reuptake inhibitor (SSRI) antidepressant that is generally
considered safe to use during pregnancy. However, the decision to continue or discontinue Prozac
should be made in consultation with the patient's healthcare provider, based on the individual
patient's situation and needs.
Famotidine is a medication used to reduce stomach acid production, and vitamin B6 is a dietary
supplement often used to alleviate nausea during pregnancy. Both are generally considered safe to
use during pregnancy when taken as directed.
A. Heartburn
B. Constipation
C. Nausea and vomiting
D. Minimize risk of hemorrhoids
Answer: C
B6 (pyridoxine) is commonly used to help manage morning sickness/nausea and vomiting associated
with pregnancy.
A 55-year-old male with a history of atrial fibrillation, CHF, is diagnosed with tuberculosis. His primary
provider places him on rifampin and isoniazid, ethambutol, and pyrazinamide. His other medications
include Coumadin, Lasix, lisinopril, and metoprolol.
A. Kidney failure
B. Hepatitis
C. Thrombocytopenia
D. Increased risk of pancreatic cancer
Answer: B
Isoniazid is notoriously known for hepatitis (sometimes fatal).
A. Hypomagnesemia
B. Hypokalemia
C. Hyponatremia
D. All of the above can result due to Lasix use
Answer: D
Lasix (furosemide) can cause a decrease in all these electrolytes. Lasix can also cause hypocalcemia.
113. When this patient has their rifampin discontinued, what will happen to Coumadin concentrations
and INR?
Answer: A
In this case, rifampin is a strong enzyme inducer. When this medication is initiated, it will likely cause
a reduction in the concentration of Coumadin and a resulting drop in INR. When the drug is
discontinued, Coumadin concentrations will likely go up, and the INR will follow.
114. Which of the following medications used for HIV has a warning for lactic acidosis and severe
hepatomegaly?
A. Efavirenz
B. Abacavir
C. Lopinavir
D. Ritonavir
Answer: B
Abacavir is a nucleoside reverse transcriptase inhibitor (NRTI) that has a warning for lactic acidosis
and hepatomegaly. Lopinavir and ritonavir are protease inhibitors, with ritonavir used as a
pharmacokinetic booster. Efavirenz is an non-nucleoside reverse transcriptase inhibitor (NNRTI).
A 69-year-old male recently fell at home and presents to the ER with significant back pain. He states that
he took a couple of Tylenol before he came, but it hasn’t done a thing to help his pain. Past medical
history includes hypertension, mild CHF, and epilepsy. Current medications include Keppra, Dilantin,
clonidine, and Zestril.
115. What would be the most appropriate recommendation to treat his acute pain?
A. Fentanyl patch
B. Oxycontin
C. Tramadol
D. Morphine
Answer: D
Fentanyl patches are not meant for acute pain. Fentanyl patches take a long time to begin their
analgesic properties. Oxycontin is a long acting product which is not the optimal choice for this
patient. Tramadol would be a potential option, but the seizure history would discourage us from
using it. Morphine would be the best choice given the options.
116. If we used an NSAID in the above case, which of the following should be limited to use no longer
than 5 days?
A. Ketorolac
B. Ibuprofen
C. Nabumetone
D. Piroxicam
Answer: A
Ketorolac should be limited to 5 days due to GI bleed risk.
117. Which of the following vitals would be important in assessing fall risk in this patient?
A. Blood pressure
B. Dilantin Level
C. Timing and location of fall
D. All of the above
Answer: D
All of the above should be assessed as well as fall history, recent medication changes, etc.
Dilantin (phenytoin) can cause impaired coordination and drowsiness, which can contribute to fall
risks.
A. 2,225 mg
B. 1,125 mg
C. 1,425 mg
D. 1,550 mg
Answer: C
165 centimeters is about 5 feet 5 inches.
This is a female patient, so the female ideal body weight equation should be used: 45.5 kg + (2.3 kg x
5) = 57 kg
57 kg x 25 mg/kg = 1,425 mg
119. How many vials of 80 mg/2 mL (2 mL vials) tobramycin would be required if a provider asks for a 6
mg/kg dose for a female patient weighing 189 lbs?
Answer: 7 vials
First, figure out the patient’s weight in kilograms: 189 lbs/2.2 kg = 85.7 kg
Then figure out how many mg of tobramycin is required: 85.7 kg x 6 mg/kg = 514 mg
Then use the concentration to figure out the required volume: 80 mg/2 mL = 40 mg/mL; therefore,
514 mg/(40 mg/ml) = 12.85 mL of tobramycin solution
12 mL is insufficient. Each vial is 2 mL, so the next lowest volume available is 14 mL. 14 mL x (1 vial/2
mL) = 7 vials are required to fill this order as is.
120. A patient presents a prescription for an infant. The order calls for 15 grams of 0.3% hydrocortisone
cream. The pharmacy has a stock 1% hydrocortisone cream and cold cream.
How much hydrocortisone 1% cream (in grams) is needed to make 15 grams of the 0.3% cream?
Round to the nearest tenth decimal place.
First figure out how much hydrocortisone is needed: 15 grams of the 0.3% product corresponds to
[15 g x (0.3 g/100 g)] = 0.045 grams or 45 mg of hydrocortisone.
In each gram of hydrocortisone 1% contains 10 mg of hydrocortisone (or 0.01 grams), and 0.045 g of
hydrocortisone is required. Therefore, 0.045 g hydrocortisone x (1 g/0.01 g) = 4.5 grams (correct
answer) of 1% hydrocortisone is needed for the final product.
If the question were asking for the quantity of cold cream, subtract 4.5 g of hydrocortisone 1% from
the 15 g total to get to a final quantity of 10.5 grams.
Approximately how many hours will it take for this bag to be used up? Round to the nearest hour.
Answer: 11 hours
Converting 3 mg/min to flow rate in mL/min: 3 mg/min x (1 mL/8 mg) = 0.375 mL/min
Use the flow rate and the volume of the bag (250 mL) to calculate the answer in minutes: 250 mL x (1
min/0.375 mL) = 667 minutes.
A 69-year-old female patient has a history of kidney disease, DVT, hypertension, neuropathy, and back
spasms, currently receiving the following medications: Coumadin, metoprolol, gabapentin,
cyclobenzaprine, and Cymbalta.
She presents to the pharmacy with her recent lab values as well:
● Potassium = 4.5 mEq/L
● Sodium = 138 mEq/L
● Chloride = 106 mEq/L
● Serum Creatinine = 1.9 mg/dL
● Weight = 156 lbs
● INR = 2.1
● Hemoglobin = 14.1 g/dL
A. Coumadin
B. Gabapentin
C. Cyclobenzaprine
D. Cymbalta
Answer: B
Gabapentin only undergoes minimal metabolism and is eliminated primarily via the kidneys.
Gabapentin dosing should be watched closely in a patient with a history of kidney disease, especially
with changes in kidney function, as gabapentin can accumulate.
Answer: B
As the dose of gabapentin is increased, a lower overall percentage of that dose will be absorbed due
to transporter saturation. This leads to a proportionally smaller increase in concentration compared
to increase in dose.
Answer: A
Option A describes cyclobenzaprine. Option C describes benzodiazepines. Option D describes
Lidoderm (lidocaine).
A. Spironolactone
B. Hydrocortisone
C. Avoid all corticosteroids as they can cause medication-induced adrenal insufficiency
D. Propranolol
E. Phenytoin for seizure prophylaxis
Answer: B
Addison’s disease is characterized by a lack of endogenous cortisol and other similar precursors. It is
treated with exogenous steroids like hydrocortisone.
Propranolol, phenytoin, and spironolactone are not used in the treatment of Addison’s. It may help
to remember that Addison’s is essentially the opposite of Cushing’s, which can be caused by
excessive, long term doses of corticosteroids.
126. A patient presents to the pharmacy with a new prescription for Zyban to help with smoking
cessation. She has a history of high blood pressure, smoking, multiple sclerosis, seizures, and anxiety.
Which disease state could be significantly exacerbated by this order?
A. MS
B. High blood pressure
C. Anxiety
D. Seizures
Answer: D
Although other conditions listed may also potentially be affected on rare occasions, Zyban
(bupropion) is known to lower seizure threshold (i.e., increases the risk of having a seizure).
Therefore, this medication should not be used.
A. Vancomycin
B. Dicloxacillin
C. Ceftriaxone
D. Zosyn
Answer: B
According to the 2014 IDSA Guidelines for Skin and Soft Tissue Infection, for mild non-purulent cases
of cellulitis the use of oral antibiotics with MSSA coverage (such as dicloxacillin) are recommended.
The other agents would not be appropriate for this mild case of cellulitis.
128. Which of the following ethnic groups is potentially at a higher risk of Stevens Johnson Syndrome
(SJS) from Tegretol?
A. Asians
B. White females
C. Blacks
D. Hispanics
Answer: A
The risk of Stevens Johnson Syndrome (SJS)/toxic epidermal necrolysis (TEN) from carbamazepine is
significantly increased in patients positive for the HLA-B*1502 allele. This allele is strongly associated
with ancestry across broad areas of Asia, including South Asian Indians.
129. Which of the following is generally not true in regards to changes in pharmacokinetics in older adults
(i.e., 65+ years)?
Answer: A
Older adults tend to experience an increase in fat composition (and decreases in muscle mass) which
may cause an increase in volume of distribution for lipophilic medications. Both kidney function and
albumin levels tend to decline with increased age.
Age generally does not affect conjugation (phase 2 reactions) via the liver. Remember that "LOT"
(lorazepam/oxazepam/temazepam) undergo phase 2 reactions and are less likely to accumulate in
older adults.
130. Which of the following is not an important side effect to monitor for with amphotericin B?
A. Infusion reaction
B. Nephrotoxicity
C. Neurotoxicity
D. Chills
Answer: C
Neurotoxicity is not a likely side effect of Amphotericin B.
A. Direct cost
B. Diet changes can impact blood concentrations
C. More drug interactions
D. Twice daily dosing for some
E. Inability to assess for adherence through blood levels
Answer: A, D and E
DOACs are generally more expensive than warfarin, and some require twice daily dosing (apixaban,
dabigatran). Unlike warfarin, which can be routinely monitored for adherence by measuring INR,
there currently is no effective way to assess adherence to DOACs (other than patient reports and
refill history).
A 41-year-old female has a history of hypertension, smoking, and asthma. Current medications include:
Ortho Tri Cyclen for birth control, nifedipine, varenicline, nicotine gum as needed, and albuterol as
needed. She has been complaining of abnormal dreams of late.
Answer: C
Varenicline (Chantix) is known for causing abnormal dreams.
133. Which of the following would put her at increased risk of a blood clot? Select all that apply.
A. Age
B. Ortho Tri Cyclen
C. Smoking
D. Asthma
Answer: A, B and C
The patient’s diagnosis of asthma would not put her at increased risk of a blood clot. Ortho Tri Cyclen
should be reassessed given her other risk factors (age, smoking) for clotting.
134. Which of the following is not true in regards to the use of vancomycin?
Answer: A
Oral vancomycin is poorly absorbed and is mostly confined to the GI tract. As such, it is usually only
used to treat C. difficile infections.
Answer: B
A. Indomethacin
B. Diclofenac
C. Meloxicam
D. Sulindac
Answer: B
Diclofenac (Voltaren) is available as a topical NSAID product.
137. A patient is switching from Dilantin (total daily dose of 300 mg) daily extended release capsules to
the liquid suspension (total daily dose of 300 mg).
Which of the following would be an incorrect educational point on the use of Dilantin?
Answer: B
Patients with low albumin levels have higher fraction of unbound phenytoin (Dilantin) and are
therefore at a higher risk of phenytoin toxicity, not seizure. Recall that there is a correction method to
account for this difference.
A. Grapefruit juice
B. Erythromycin
C. Diflucan
D. St. John’s Wort
Answer: D
St. John’s Wort is an inducer of CYP3A4, not an inhibitor.
A. COMT inhibitor
B. MAOI
C. Dopamine Agonist
D. Acetylcholinesterase Inhibitor
Answer: A
Comtan (entacapone) is an inhibitor of catechol-O-methyltransferase (COMT), an enzyme that
metabolizes levodopa. Used as an adjunct in the treatment of Parkinson’s disease, it helps sustain
levodopa levels when used in combo with Sinemet.
A. Carafate
B. Reglan
C. Compazine
D. Omeprazole
Answer: B
Reglan (metoclopramide) is indicated for gastroparesis (a slowing of the gut transit time). A unique
side effect of Reglan is that it can block dopamine, causing unwanted movement-related effects,
especially in patients with preexisting movement disorders involving a lack of dopamine (e.g.,
Parkinson’s disease).
141. Which of the following would likely exacerbate IBS with predominant diarrhea?
A. Dicyclomine
B. Lomotil
C. Elavil
D. Zoloft
Answer: D
Zoloft (sertraline, an SSRI) is very serotonergic and can increase the risk of exacerbating loose stools.
Elavil (amitriptyline, a TCA) is highly anticholinergic, and is therefore constipating. Dicyclomine (an
anticholinergic) can actually be used to treat IBS. Lomotil (diphenoxylate-atropine), a combination of
a mu-opioid receptor agonist and an anticholinergic, is constipating as well.
142. A patient presents to the pharmacy not having taken his medications for two days as he forgot them
when he left for vacation. Which of the following would be the highest risk to suddenly stop?
A. Metoprolol
B. Tylenol
C. Xalatan
D. Lipitor
Answer: A
Abrupt discontinuation of beta blockers can potentially cause significant rebound
hypertension/tachycardia.
A. Methimazole
B. Prednisone
C. Hydroxyzine
D. Prochlorperazine
Answer: A
Graves’ disease causes hyperthyroidism. Signs and symptoms of this disease include tachycardia,
weight loss, increased osteoporosis risk, insomnia, and anxiety. Methimazole and propylthiouracil
(PTU) are two medications that can treat Graves’ disease. Beta blockers can be used for the
treatment of acute thyroid storms.
A. Parkinson’s disease
B. Asthma
C. Multiple sclerosis
D. Diabetes
Answer: B
Methacholine is a cholinergic agonist which causes the airways to close up, making it difficult for
patients with asthma to breathe.
Remember that inhaled anticholinergics like ipratropium are used to help open up the airways.
145. Which of the following antibiotics has poor lung penetration and would greatly increase the risk of
rhabdomyolysis?
A. Clindamycin
B. Daptomycin
C. Bactrim
D. Gentamicin
Answer: B
Daptomycin is indicated for skin and soft tissue infections, has activity against MRSA, poor lung
penetration, and will increase the risk of rhabdomyolysis, especially with statin use.
146. For which of the following patients would a pneumococcal vaccine not need to be given?
Answer: A
CDC recommends pneumococcal vaccination for all adults 65 and older.
CDC also recommends pneumococcal vaccination for adults 19 through 64 who have chronic lung
disease, including asthma and COPD.
A. Cohort study
B. Case control study
C. Randomized controlled trials
D. Case study
Answer: D
Individual case studies are the lowest on the strength of evidence scale. Randomized controlled trials
(RCTs) are the strongest evidence-based medicine studies – hence this is what is required for new
drugs. RCTs are often difficult to conduct with rare diseases due to difficulty in enrolling enough
patients to properly power the study.
148. The MedWatch program collects data from which of the following phase(s) of clinical trials?
A. Phase 1
B. Phase 2
C. Phase 3
D. Phase 4
Answer: D
MedWatch collects information from post marketing surveillance (also sometimes called phase 4).
Phase 3 is the last phase of study before a medication is approved and released to the market.
149. A 68-year-old female with a history of epilepsy and diabetes is currently receiving the following:
● Phenytoin
● Carbamazepine
● Pioglitazone
● Metformin
Recent labs revealed hyponatremia. Which medication is the most likely cause?
A. Phenytoin
B. Carbamazepine
C. Pioglitazone
D. Metformin
Answer: B
Carbamazepine has the potential to cause hyponatremia.
Answer: B
The FDA-approved use of tPA (alteplase) for acute ischemic stroke is within 3 hours of symptom
onset. Guidelines advise against using tPA (specifically, alteplase) for acute ischemic stroke in patients
taking warfarin whose INR is greater than 1.7. Waiting for INR to fall below 1.7 is not an appropriate
option as it will likely be too late at that point to administer tPA.
Answer: A
A type I error occurs when the null hypothesis (no effect/difference between groups) is rejected
when it is actually true (aka false positive). The p-value helps determine the likelihood of a type I
error occurring.
The probability of a type II error (aka false negative) is not directly represented by the p-value.
The p-value does not indicate clinical significance. It only indicates statistical significance. Clinical
significance relates to the practical importance or real-world relevance of the findings.
The ability of a trial to detect a difference that truly exists is related to the concept of power, not
p-value.
152. A 46-year-old female is assessed with right knee pain. She was instructed a couple of weeks ago to
try capsaicin cream for mild to moderate pain. Past medical history includes GI bleed and chronic
kidney disease. She has tried it a few times when her knee was aching but reports that it has not
been effective. What is the most appropriate recommendation?
A. Keep using the capsaicin, but use at least three times daily for at least a few weeks
B. Self treatment is not recommended at this point
C. Recommend ibuprofen
D. Recommend glucosamine/chondroitin
Answer: A
Capsaicin should be applied on a consistent basis (3-4 times daily) to deplete substance P and
produce analgesic effects. Capsaicin may take 2-4 weeks for effective pain relief. Ibuprofen is not
recommended due to the patient’s history of GI bleed and CKD. There is currently mixed evidence for
the effectiveness of glucosamine/chondroitin for relief of knee pain.
A. Observational bias
B. Selection bias
C. Recall bias
D. Interviewer bias
Answer: B
Bias in the selection or choosing of participants in a study is selection bias.
154. Variables X and Y have a correlation coefficient of -0.0024. What does this mean?
Answer: A
Correlation coefficient ranges from +1 to -1, with +1 being complete positive correlation and -1 being
complete inverse correlation. The given value is close to 0, which translates to very little correlation.
A. 25/75 plasma/interstitial
B. 50/50 plasma/interstitial
C. 60/40 plasma/interstitial
D. None of the above
Answer: A
Extracellular fluid volume makes up about a third of the total body water. 75% of the extracellular
fluid volume is found in the interstitial space, and the remaining 25% is found in the plasma.
A. Levofloxacin
B. Moxifloxacin
C. Clarithromycin
D. Erythromycin
Answer: D
Due to its well known prokinetic properties, erythromycin can be used (off-label) for the
management of gastroparesis.
Labs:
● Blood glucose = 555 mg/dL
● Sodium = 134 mEq/L
● Potassium = 3.0 mEq/L
● Creatinine = 1.6 mg/dL
● pH = 7.23
A. Initiate insulin
B. Initiate IV potassium replacement
C. Initiate oral hydration
D. Initiate sodium bicarbonate to raise pH
Answer: B
With ketoacidosis, the key is to correct the underlying hyperglycemia. However, if the patient is
already hypokalemic with K < 3.3 mEq/L, potassium should be corrected before adding insulin (insulin
can cause an intracellular shift of potassium and lead to profound hypokalemia). Hydration is
recommended using intravenous saline 0.9% or 0.45%. Bicarbonate replacement is recommended in
patients with pH < 6.9.
158. A 55-year-old male has been hospitalized for about 7 days following a recent hip surgery. He starts to
develop increased coughing and a temperature of 38.2 degrees Celsius and is diagnosed with
pneumonia. What is the most appropriate empiric pneumonia therapy?
Answer: C
With no MRSA risk factors or mortality risk mentioned in the prompt, the best option is one
guideline-recommended agent that covers Pseudomonas and MSSA.
159. A 36-year-old male present with intractable hiccups. What medication is indicated for the treatment
of this?
A. Lorazepam
B. Levetiracetam
C. Propranolol
D. Chlorpromazine
Answer: D
Chlorpromazine is indicated for the treatment of intractable hiccups.
Answer: D
Fluoroquinolones bind to and inhibit topoisomerase II and IV (not I and III).
161. In oncology, which one of the following medication regimens is least likely to cause chemotherapy
induced nausea and vomiting?
A. Dacarbazine
B. Doxorubicin (< 60 mg/m2)
C. Cisplatin
D. Vincristine
Answer: D
Dacarbazine and cisplatin are considered high emetic risk regimens. Doxorubicin is considered a
moderate emetic risk regimen. Vincristine is considered to have minimal emetic risk.
162. A mother presents to the pharmacy with questions on different pain medications for her 8 month
old infant. She believes her baby is teething and would like something to help with the pain. She will
not use Tylenol as she is concerned that it will cause liver toxicity in her baby.
A. Aspirin
B. Ibuprofen
C. Tramadol
D. None of the above
Answer: B
Ibuprofen would be an acceptable alternative to Tylenol in pediatrics for the management of pain
and/or fever. There are infant suspensions available for the patient’s age group. Aspirin should not be
used over the counter in children younger than 12 years.
A. Symbicort
B. DuoNeb
C. Spiriva
D. Daliresp
Answer: A
Based on the prescription and the symptoms, this patient likely has thrush. Symbicort contains an
inhaled corticosteroid, and patients should be counseled to rinse their mouth after each use in order
to avoid thrush.
A 45-year-old male has a history of hyperlipidemia, gout, hypertension, and rheumatoid arthritis.
A. Lipitor
B. Niacin
C. Methotrexate
D. Lisinopril
Answer: B
Niacin is notorious for flushing symptoms. Niacin can also cause elevated uric acid levels and
potentially exacerbate gout
Answer: A
● Allopurinol inhibits xanthine oxidase
● Probenecid increases renal excretion of uric acid
● Colchicine inhibits beta tubulin polymerization
● NSAIDs reduce inflammation via COX inhibition
166. Which of the following is a notable drug interaction that can occur when Lipitor and niacin are taken
together?
Answer: B
Concomitant use of niacin and atorvastatin can increase the risk of myopathy, especially during the
initial months of therapy and any periods of upward dosage titration.
Answer: A
As sample size increases, the power to detect a difference between the two groups increases as well.
Type 2 error is not detecting a difference when there is one; type 1 error is detecting a difference
when there isn’t one in reality. Bias is more prevalent in type 1 errors. Intention-to-treat analysis
considers all randomized subjects. P-value threshold of 0.05 is used to determine statistical
significance, not clinical.
Answer: D
Epinephrine is a sympathomimetic agent that has both alpha- and beta-receptor activities.
Epinephrine causes hypertension by inducing vasoconstriction.
A. Inhibits calcineurin
B. Inhibits inosine 5’-monophosphate dehydrogenase
C. Inhibits T-lymphocyte activation by binding to FKBP-12
D. Binds to intracellular receptors and affect gene expression to reduce inflammation
Answer: A
Options B, C, and D correspond to mycophenolate mofetil, tacrolimus, and corticosteroids,
respectively.
170. An over the counter medication (solution) is 0.7% active ingredient. How many mg would be in a 30
cc bottle? Round to the nearest whole mg.
Answer: 210 mg
Therefore, 0.7% indicates 0.7 grams of the active ingredient (700 mg) per 100 mL.
The total volume given in the prompt is 30 mL (1 cc=1 mL).
30 mL x (700 mg/100 mL)=210 mg of the active ingredient.
171. What is the primary mechanism of action of olanzapine in the treatment of schizophrenia?
Answer: D
Antipsychotics generally block dopamine receptors as their primary mechanism. Olanzapine blocks
D2 dopamine receptors and 5HT-2A serotonin receptors.
172. A 48-year-old male presents with an acute exacerbation of his rheumatoid arthritis. He is currently
on methotrexate 7.5 mg weekly.
Which of the following would be most appropriate to acutely relieve the pain symptoms?
A. Increase methotrexate
B. Add Tylenol
C. Add ibuprofen
D. Switch to hydroxychloroquine
Answer: C
Switching to another DMARD or increasing methotrexate may be considered long-term, but these
actions will likely not relieve immediate symptoms. NSAIDs are preferred over Tylenol for managing
rheumatoid arthritis because acetaminophen has no effect on inflammation.
A 134 kg patient with chronic kidney disease has an order written for Aranesp 0.45 mcg/kg once weekly.
The concentration in stock at the pharmacy is 25 mcg/0.42 mL.
173. How many mL will be required to fill this order? Round to the nearest hundredth decimal place.
Answer: 1.01 mL
Starting with the patient’s weight: 134 kg x 0.45 mcg/kg = 60.3 mcg.
Converting the total dose to mLs using the given concentration:
60.3 mcg x (0.42 mL / 25 mcg) = 1.01 mL.
Answer: C
Aranesp (darbepoetin alfa) is an erythropoiesis-stimulating agent (ESA). This agent is used to treat
anemia and increase red blood cell production. This agent can increase the risk of serious
cardiovascular events, especially when the hemoglobin level is greater than 11 g/dL. Aranesp can
cause an increase in blood pressure, but this is not part of the boxed warning.
175. Which one of the following medications could be used in atrial fibrillation and hypertension?
A. Terazosin
B. Calan
C. Amlodipine
D. Lisinopril
Answer: B
Calan (verapamil) is a non-dihydropyridine calcium channel blocker. It can slow down the ventricular
rate in atrial fibrillation treatment.
Amlodipine is a dihydropyridine calcium channel blocker, which has greater selectivity for vascular
smooth muscle cells than cardiac muscle cells (and therefore has no established use in atrial
fibrillation).
Terazosin is an alpha blocker, and lisinopril is an ACE inhibitor - neither of these agents are used to
treat atrial fibrillation.
A. Metronidazole
B. Carbamazepine
C. St. John’s Wort
D. Rifampin
Answer: A
Metronidazole, quinolones, and Bactrim are antibiotics that can raise INR when concomitantly used
with warfarin. Carbamazepine, St. John’s Wort, and rifampin certainly can interact with Coumadin,
but they are enzyme inducers and would cause INR to go down when initiated or increased.
177. Which of the following medications has the potential to cause both hyponatremia and
hyperkalemia?
A. Enalapril
B. Hydrochlorothiazide
C. Metoprolol
D. Aldactone
Answer: D
Enalapril can cause hyperkalemia but generally has no effect on sodium. Metoprolol is a beta blocker
and generally does not affect electrolytes. Hydrochlorothiazide can cause both hyponatremia and
hypokalemia.
A. Dopamine agonist
B. NMDA receptor antagonist
C. Acetylcholinesterase inhibitor
D. Neuraminidase Inhibitor
Answer: B
Memantine (Namenda) is an NMDA receptor antagonist used in the treatment of dementia.
Dopamine agonists used to treat Parkinson’s disease include ropinirole and pramipexole.
Acetylcholinesterase inhibitors used for dementia include rivastigmine and donepezil. An example of
a neuraminidase inhibitor is oseltamivir, which is used for prophylaxis and treatment of influenza.
A. Circulation problems
B. Increased oral secretions
C. Hypertension
D. Growth suppression
Answer: B
Vyvanse (lisdexamfetamine) is a CNS stimulant. It can cause dry mouth, not excessive salivation.
Her current diagnoses include insomnia, constipation, pain, hypertension, xerostomia, and
keratoconjunctivitis sicca.
180. She has a very busy schedule and would like to do all her eye drops at one time. What is the most
appropriate recommendation?
Answer: C
5 minutes is the recommended separation between eye drops when administering more than one.
181. Which of the following medications is likely to exacerbate her xerostomia and keratoconjunctivitis
sicca?
A. Oxycodone
B. Diltiazem
C. Restasis
D. Doxepin
Answer: D
Doxepin is an older TCA and has anticholinergic properties. Of the given choices, doxepin is the most
likely to exacerbate her xerostomia (dry mouth) and keratoconjunctivitis sicca (dry eyes).
Answer: B
Lumigan (bimatoprost) is a prostaglandin analog used in the treatment of glaucoma. Timoptic
(timolol) is an example of a beta-blocker, and Alphagan (brimonidine) is an example of an alpha-2
agonist. Corticosteroids are not used for the treatment of glaucoma, but for inflammation of the eye.
183. A 64-year-old female is receiving Oxycontin 80 mg twice daily for the management of back pain. She
has been taking this medication for years, but has had some difficulty swallowing recently. The
provider would like to switch Oxycontin to morphine oral solution.
What would be the most appropriate equivalent conversion from Oxycontin 80 mg twice daily?
Answer: A
Oxycodone’s morphine equivalent conversion factor is 1.5:
184. Which of the following is not an advantage of using Buspar over a benzodiazepine to treat anxiety?
Answer: A
For the treatment of anxiety, Buspar is similar to SSRIs in that it generally takes some time to be
effective. Buspar will likely not work as well as benzodiazepines when used as needed.
Oseltamivir Prophylaxis
< 15 kg = 30 mg once daily for 10 days
15.1-23 kg = 45 mg once daily for 10 days
23.1-40 kg = 60 mg once daily for 10 days
≥ 40.1 kg = 75 mg once daily for 10 days
Oseltamivir Treatment
< 15 kg = 30 mg BID for 5 days
15.1-23 kg = 45 mg BID for 5 days
23.1-40 kg = 60 mg BID for 5 days
≥ 40.1 kg = 75 mg BID for 5 days
Answer: A
The child’s body weight in kilograms is 45 lb x (1 kg/2.2 lb) = 20.45 kg
For this body weight, the appropriate treatment dose is 45 mg BID for 5 days.
Answer: D
Oseltamivir requires dose adjustments for impaired renal function.
An elderly gentleman has recently had cataracts removed from his eyes and still has complaints of visual
problems.
187. Which medication is most likely responsible for contributing to his cataracts?
A. Prednisone
B. Ramipril
C. Methotrexate
D. Terazosin
Answer: A
Long term use of corticosteroids like prednisone is one of the most common causes of drug induced
cataracts.
188. Which of the following is the appropriate route of delivery for etanercept?
A. Oral
B. Intra-articular
C. IV infusion
D. Subcutaneous
Answer: D
Etanercept is administered via subcutaneous injection.
A physician orders a 3 mcg/kg/min drip of Drug A. This drug is typically put into a 250 mL bag of D5W. The
patient’s weight is 135 pounds.
189. How much Drug A in mg will this patient receive in 30 minutes? Round to the nearest tenth decimal
place.
Answer: 5.5 mg
190. If the infusion ran continuously for 48 hours at the rate of 3 mcg/kg/min, how many bags of D5W,
each containing 200 mg of Drug A, would the patient need?
Answer: 3 bags
At 200 mg per bag, 3 bags of 200 mg D5W would be required to fulfill this order.
Answer: B
The Cockcroft-Gault equation for estimating creatinine clearance (CrCl) is as follows:
eCrCl (mL/min) = [ 140-age) x (lean body weight in kg) x (0.85 if female) / (SCr in mg/dL) ] x 72
All of the above are true except for "B" - a higher lean body weight will yield a higher estimated
creatinine clearance.
With an increase in shortness of breath, JS presents to the emergency department. He has noticed that
swelling around his ankles has increased some over the last couple of days. He also reports frequent OTC
naproxen use because of a sore back.
His other diagnoses include COPD, GERD, history of myocardial infarction, heart failure, and atrial
fibrillation.
192. Which of the following monitoring parameters would be utilized to assess this patient for cardiac
conduction abnormalities?
A. BNP
B. Chest X-Ray
C. Troponin
D. ECG
Answer: D
An ECG (electrocardiogram) would be utilized to assess this patient for cardiac conduction
abnormalities. An ECG is a non-invasive test that records the electrical activity of the heart and can
detect abnormalities in heart rhythm and conduction, such as arrhythmias, heart block, and other
cardiac abnormalities.
BNP (brain natriuretic peptide) is a blood test that measures levels of a hormone released by the
heart in response to changes in pressure or volume, and it is typically used to diagnose and monitor
heart failure.
Chest X-ray is an imaging test that uses radiation to produce images of the heart and lungs, and it is
often used to diagnose and monitor various cardiac and pulmonary conditions, such as pneumonia,
congestive heart failure, and pulmonary edema.
Troponin is a blood test that measures levels of a protein released into the bloodstream when the
heart muscle is damaged, and it is used to diagnose and monitor acute coronary syndromes, such as
heart attacks.
While these monitoring parameters may be useful in assessing cardiac function, they are not specific
to detecting cardiac conduction abnormalities, which is best evaluated with an ECG.
● SpO2 = 92%
● Pulse = 98 BPM
● BP = 152/88 mmHg
● Respiratory rate = 36 resp/min
● BNP = 986 pg/mL
A. Nitroprusside
B. Digoxin
C. Labetalol
D. Torsemide
Answer: D
With the elevated BNP (≥ 100 pg/mL) and symptoms of fluid overload, this patient is likely having an
episode of acute heart failure decompensation. In most cases, rapid relief of congestion and
reduction of fluid overload are the primary goals of initial therapy for acute heart failure
decompensation.
Loop diuretics (such as furosemide) act quickly to reduce fluid overload and congestion, often within
hours of administration, making them the most critical agent to administer first during acute heart
failure hospitalization.
The ambulance arrives at the emergency department with a 35-year-old male who was just in a car
accident. Because of the injuries to his chest, he is having an incredibly difficult time breathing and the
provider would like to intubate him.
194. Which of the following agents should be selected to ensure the most rapid intubation?
A. Dantrolene
B. Cisatracurium
C. Succinylcholine
D. Intrathecal baclofen
Answer: C
Succinylcholine is the agent that should be selected to ensure the most rapid intubation.
Succinylcholine is a depolarizing neuromuscular blocker that works quickly and has a rapid onset of
action. It is commonly used for rapid sequence intubation in emergency situations.
Intrathecal baclofen is a medication used to treat spasticity, and it is not used for intubation.
Which of the following agents would be utilized as a reversal agent for pancuronium?
A. Pilocarpine
B. Succinylcholine
C. Fluorouracil
D. Neostigmine
Answer: D
Neostigmine would be utilized as a reversal agent for pancuronium. Neostigmine is a medication that
works by inhibiting acetylcholinesterase, an enzyme that breaks down acetylcholine. This leads to an
increase in acetylcholine levels, which can reverse the effects of non-depolarizing neuromuscular
blockers like pancuronium. Succinylcholine, a depolarizing neuromuscular blocking agent, does not
have a specific reversal agent. However, the effects of succinylcholine are relatively short-lived, and
the medication is rapidly metabolized by plasma cholinesterase, an enzyme in the blood. As a result,
the muscle paralysis caused by succinylcholine typically resolves within a few minutes, and the
patient will begin to regain muscle function on their own without the need for a reversal agent
196. Which of the following antihypertensive agents are considered safe for use during pregnancy?
A. Methyldopa
B. Labetalol
C. Nifedipine
D. All of the above
Answer: D
Methyldopa, labetalol and nifedipine are considered safe for use during pregnancy when
antihypertensive therapy is necessary.
Jane is a 34-year-old female. She presents very excitedly as she recently found out she is pregnant after
years of trying. She has frequent headaches, about 4-6 per month, that range in severity. She is currently
on prophylaxis for headaches and also has severe depression.
A. Sertraline
B. Losartan
C. Depakote
D. Nifedipine
Answer: B and C
Depakote (divalproex) and losartan are known for their potential to cause birth defects and should be
discontinued after alternative therapies are identified.
Nifedipine, sertraline and TUMS (as-needed) are considered safe for use during
pregnancy. Depakote (divalproex) has been associated with an increased risk of birth defects, such as
neural tube defects, when used during pregnancy. Losartan has also been associated with fetal
toxicity and should generally be avoided during pregnancy. Therefore, both medications are typically
discontinued when alternative therapies are identified, especially in females who are pregnant or
planning to become pregnant.
Nifedipine and TUMS are generally considered safe for use during pregnancy. Sertraline, a selective
serotonin reuptake inhibitor (SSRI) antidepressant, is also considered safe for use during pregnancy.
198. Jane is complaining of a migraine headache today and is wondering what she could take. Which is
the safest initial option for abortive migraine therapy?
A. Acetaminophen
B. Ibuprofen
C. Sumatriptan
D. Fioricet
Answer: A
Acetaminophen is generally considered the safest initial option for abortive migraine therapy during
pregnancy. Ibuprofen and other nonsteroidal anti-inflammatory drugs (NSAIDs) are generally not
recommended during pregnancy, especially during the third trimester, due to their potential to cause
complications in both the mother and the developing fetus.
Sumatriptan is a prescription medication that is specifically designed for the treatment of migraines.
While it has been used in pregnancy, its safety during pregnancy has not been fully established, and it
is generally reserved for cases where acetaminophen is not effective.
Fioricet is a combination medication that contains acetaminophen, butalbital, and caffeine. While
acetaminophen is generally considered safe during pregnancy, butalbital and caffeine are not
recommended due to their potential to cause fetal harm.
A. Captopril
B. Spironolactone
C. Nitroprusside
D. Labetalol
Answer: D
Labetalol would be the best choice for immediate reduction of blood pressure and for the safety of
the fetus in a pregnant patient with a hypertensive emergency. Labetalol is a medication that is
commonly used in the management of hypertension during pregnancy. It works by blocking both
alpha and beta adrenergic receptors, which leads to a decrease in blood pressure.
Captopril and other angiotensin-converting enzyme (ACE) inhibitors are generally not recommended
during pregnancy due to their potential to cause fetal harm.
200. A 58-year-old male is rushed to your emergency department with complaints of chest pain. He is
diagnosed with unstable angina. Which of the following assessments would be most appropriate to
determine the likelihood of a cardiac ischemic event and help determine the urgency for further
intervention?
A. CHADS2-Vasc
B. TIMI
C. PHQ-9
D. CURB-65
Answer: B
The TIMI score (Thrombolysis In Myocardial Infarction) would be the most appropriate assessment in
this scenario to determine the likelihood of a cardiac ischemic event and help determine the urgency
for further intervention.
The PHQ-9 (Patient Health Questionnaire-9) is a screening tool used to assess the severity of
depression in adults. CURB-65 is a clinical prediction rule used to assess the severity of
community-acquired pneumonia (CAP) in adults and to determine the need for hospitalization.
Currently, the Infectious Diseases Society of America (IDSA) and the American Thoracic Society (ATS)
recommend using the Pneumonia Severity Index (PSI) as the preferred clinical prediction tool for
community-acquired pneumonia, rather than the CURB-65 score.
201. Which of the following is the most appropriate initial urate-lowering therapy for a patient with a
history of recurrent gout attacks?
A. Allopurinol
B. Febuxostat
C. Probenecid
D. Colchicine
Answer: A
Allopurinol is the most appropriate initial urate-lowering therapy for a patient with a history of
recurrent gout attacks. Allopurinol is a xanthine oxidase inhibitor that reduces the production of uric
acid in the body, making it a useful first-line medication for the management of gout.
Febuxostat is another xanthine oxidase inhibitor that can be used if allopurinol is not well-tolerated
or if there is an inadequate response to allopurinol therapy.
Probenecid is a uricosuric medication that increases uric acid excretion by the kidneys. It is generally
not used as a first-line therapy because it can be less effective than xanthine oxidase inhibitors and
has a higher risk of adverse effects.
Colchicine is not a urate-lowering therapy but is often used for the treatment of acute gout attacks or
for prophylaxis against gout flares during initiation of urate-lowering therapy.
Answer: D
Focusing on high risk medications and finding ways to minimize errors with these medications is very
important and a strong emphasis should be placed on looking at processes surrounding medication
dispensing, ordering, and/or administration. The other statements above are false. Omissions are the
most common type of error. Wrong patient errors are typically more high risk than medication
omissions. Punitive action for errors is typically not recommended for reducing errors and improving
safety.
Answer: C
The least effective strategy would be evaluation forms of the speaker, this does not indicate in any
way the amount or quality of knowledge retained.
204. A 20-year-old female is seeking birth control. She has been reluctant to take birth control because
her roommate complained about how it made her nauseous all the time. She is also worried about
breakthrough bleeding. She would prefer a daily pill versus any other birth control administration
method.
Answer: A
Higher doses of estrogens in birth control pills are generally associated with a higher risk of nausea as
a side effect. This is because estrogens can stimulate the production of stomach acid, which can
cause nausea and vomiting. Lower doses of estrogens are generally better tolerated. Progestin-only
pills (POPs), such as norethindrone 0.35 mg, are generally associated with more breakthrough
bleeding than combination oral contraceptives (COCs). Mirena, is a levonorgestrel-releasing
intrauterine device (IUD) that is inserted into the uterus by a healthcare provider. While it is a highly
effective form of birth control and can last for up to 5 years, it is not the patient's preferred method
of administration.
JT is a 54-year-old male with rheumatoid arthritis and hyperlipidemia. He presents with an LDL of 194
mg/dL, triglyceride level of 143 mg/dL, total cholesterol of 296 mg/dL, and HDL of 58 mg/dL. He has a
history of MI and is currently on simvastatin 20 mg daily.
Answer: E
The patient has established ASCVD (history of myocardial infarction) and elevated LDL levels.
Currently, the patient is not receiving high-intensity statin therapy (rosuvastatin 20-40 mg or
atorvastatin 40-80 mg) despite a clear indication. At this time, the most appropriate action would be
to change to atorvastatin 40 mg daily. Further intensification of lipid therapy may be necessary at
follow-up visits.
A. Leflunomide
B. Prednisone
C. Omeprazole
D. Famotidine
Answer: B
Prednisone has been shown to increase serum triglyceride levels, which can contribute to the
development of cardiovascular disease.
Omeprazole and famotidine are both medications that are used to treat gastroesophageal reflux
disease (GERD) and other acid-related conditions. They are not typically associated with elevations in
triglycerides.
Which of the following would be the most appropriate antibiotic selection for empiric treatment of
community-acquired pneumonia?
A. Ciprofloxacin
B. Clarithromycin
C. Doxycycline
D. Vancomycin
E. Metronidazole
Answer: C
Of the listed options, doxycycline is the most appropriate choice for empiric therapy. Due to the
noted macrolide resistance use of clarithromycin is not preferable. Ciprofloxacin does not provide
necessary coverage against S. pneumoniae (primary pathogen of community acquired pneumonia).
Intravenous vancomycin may be part of appropriate therapy (for MRSA coverage) when managing
the patient in the inpatient setting, but the patient does not require inpatient treatment (and oral
vancomycin would not be appropriate). Metronidazole is primarily used to treat anaerobic bacterial
infections, such as bacterial vaginosis or intra-abdominal infections, and is not effective against the
common pathogens that cause community-acquired pneumonia.
208. A 35-year-old female has a history of substance abuse. She also has major depression and is on
sertraline 150 mg daily. She has tried 200 mg per day in the past but felt too sedated on that dose.
Her PCP feels that she needs additional therapy and would like to try an atypical antipsychotic that
will have a low risk for weight gain as well as sedation.
A. Olanzapine
B. Quetiapine
C. Risperidone
D. Aripiprazole
Answer: D
Aripiprazole is an antipsychotic that will likely have the least weight gain as well as a lower risk of
sedation. It is also indicated for augmenting antidepressant therapy.
A new medication has been developed to help with the management of fever. The medication was given
to one group of patients and a placebo was given to the other group of patients. The mean reduction in
body temperature was (0.9-2.4 degrees Fahrenheit) with a confidence level of 95%.
A. Ratio
B. Nominal
C. Ordinal
D. Interval
Answer: D
Interval data is data that is measured on a numerical scale, where the differences between the values
are consistent and meaningful. However, there is no true zero point. Examples of interval data
include temperature (measured in Celsius or Fahrenheit) and IQ scores.
Ordinal data is ranked in a particular order or sequence, but the differences between the values are
not necessarily equal or consistent. Examples of ordinal data include rankings, letter grades, and
symptom scales.
Nominal data is data that is categorized into distinct groups or categories, with no inherent order or
ranking. Examples of nominal data include eye color, race, and blood type.
Ratio data is data that is measured on a numerical scale, where the differences between the values
are consistent and meaningful, and there is a true zero point. Examples of ratio data include weight,
height, and blood pressure.
210. Which of the following would be false if the data follows a normal distribution?
A. 95% of the data would fall within 2 standard deviations of the mean
B. The mean is equal to the median
C. Normal distribution classically follows a bell-shaped curve
D. The standard deviation will equal the mode
Answer: D
Answer D would be false if the data follows a normal distribution.
In a normal distribution, the mean and median are equal, so option B is true. The normal distribution
is also classically described as a bell-shaped curve, so option C is true. Additionally, about 95% of the
data falls within 2 standard deviations of the mean, so option A is also true.
However, the mode of a normal distribution is not necessarily equal to the standard deviation. In
fact, the normal distribution does not necessarily have a well-defined mode, as it is a continuous
probability distribution.
A. Yes
B. No
Answer: A
Yes, because the reduction was between 0.9 and 2.4 degrees Fahrenheit the reduction was
considered statistically significant. If the data range contained the value 0, it would not have been
considered significant.
212. After review, it was noted that the participants in the treatment group had a higher average
temperature at baseline than the placebo group. What type of bias would this most likely be?
A. Recall
B. Analytical
C. Selection
D. Observer
Answer: C
The type of bias that is most likely present in this situation is selection bias.
Selection bias occurs when the selection of participants for a study is not random, resulting in groups
that are not comparable at baseline. In this case, the participants in the treatment group were not
comparable to those in the placebo group, as they had a higher average temperature at baseline.
This could affect the results of the study, as the higher temperature could have an effect on the
outcome being measured, independent of the treatment or placebo being given.
Recall bias occurs when there is a systematic difference in the accuracy or completeness of
information that is reported by participants in different groups. Analytical bias occurs when there is a
flaw in the way data is collected or analyzed. Observer bias occurs when the observer's knowledge of
the participant's group assignment influences the way they collect or interpret data. These types of
bias are less likely to be present in this situation, as the bias in this case is related to the selection of
participants for the study.
A clinical trial has been set up and two groups were compared for reduced pain in fibromyalgia. One
group received Fibrono 25 mg daily and the other received placebo. Fibrono significantly reduced the
Somatic Symptoms Scale-8 (SSS-8) score with a p-value <0.001 compared to placebo.
A. Review article
B. Controlled clinical trial
C. Cohort
D. Case study
Answer: B
The setup of two groups receiving a treatment and placebo is a classic example of a controlled clinical
trial.
A. Ratio
B. Nominal
C. Ordinal
D. Interval
Answer: C
Ordinal data is ranked in a particular order or sequence, but the differences between the values are
not necessarily equal or consistent. Examples of ordinal data include rankings, letter grades, and
symptom scales.
Nominal data is data that is categorized into distinct groups or categories, with no inherent order or
ranking. Examples of nominal data include eye color, race, and blood type.
Interval data is data that is measured on a numerical scale, where the differences between the values
are consistent and meaningful. However, there is no true zero point. Examples of interval data
include temperature (measured in Celsius or Fahrenheit) and IQ scores.
Ratio data is data that is measured on a numerical scale, where the differences between the values
are consistent and meaningful, and there is a true zero point. Examples of ratio data include weight,
height, and blood pressure.
215. Within a research article, under which heading would the detailed description of doses used and
timeframe of the study generally be listed?
A. Discussion
B. Results
C. Conclusion
D. Methods
Answer: D
The Methods section of a clinical trial will generally contain this information.
Researchers are looking at studying the use of hydrochlorothiazide as a preventative therapy for
osteoporosis. They have set up two groups. One group will receive placebo and one will receive
hydrochlorothiazide 25 mg daily. Over the course of one year, 25 of the 950 study participants in the
hydrochlorothiazide group developed osteoporosis. In the placebo group, 33 of the 934 study
participants developed osteoporosis.
A. Yes
B. No
C. Too little information to tell
Answer: A
Yes. The p-value was less than alpha (0.05), so this result would be considered statistically significant.
217. Calculate the absolute risk reduction. Round to the nearest tenth of a percent.
Answer: 0.9%
Absolute risk reduction (ARR) = | % placebo group - % active group |
Placebo group: 33/934 = 0.035 = 3.5%
Active group: 25/950 = 0.026 = 2.6%
ARR=|3.5%-2.6%|=0.9%
218. What is the relative risk? Round to the nearest hundredth decimal place.
Answer: 0.74
Relative risk = incidence in treatment group/incidence in placebo group = 0.026/0.035 = 0.74285 =
0.74
Remember that a relative risk of 1 essentially means that there is no difference between the groups.
If the relative risk is greater than 1, it is going to indicate that the treatment group increases the risk
of something compared to placebo. If the relative risk is less than 1 it conveys that the treatment
group has a protective effect. In this case, hydrochlorothiazide demonstrates evidence of improved
protection against osteoporosis compared to placebo.
Answer: 26%
Remember that relative risk reduction will always sound a lot better than the absolute risk reduction.
Answer: 112
NNT = 1/ARR = 1/0.9% = 1/0.009 = 111.1
NNT should always be rounded up to the nearest whole number (112).
This means that one would need to treat 112 patients for one year to prevent one case of
osteoporosis.
221. A prescription arrives for 1.5% triamcinolone cream. The only available strengths are 3% and 0.5%.
How much 3% cream (in grams) will this compound require if the order is written for 30 grams?
Based on these calculations, 1 part (12 grams) of the 3% strength and 1.5 parts (18 grams) of the
0.5% strength are needed for this compound.
A. Flomax
B. Pantoprazole
C. Finasteride
D. Tramadol
Answer: C
Finasteride can cause birth defects in male fetuses. Women who are pregnant or may become
pregnant should not handle finasteride, especially when broken or crushed, due to risk of possible
exposure to a male fetus.
Funnypenem 100 mg/kg per day IV has been ordered. The infusion is ordered every 12 hours.
Funnypenem comes in 2 gram vials.
To reconstitute 2 grams of Funnypenem, it requires 6.8 mL of diluent. After reconstitution, the vial will
contain 10 mL with a concentration of 200 mg/mL. Funnypenem is usually added to a 50 mL bag of
normal saline and infused over one hour.
Answer: 3 vials
111 mg/kg/day x 100 kg = 11,100 mg daily = 5,550 mg per infusion (given twice daily)
5,550 mg per infusion requires a total of 3 two-gram vials (6000 mg total) for each dose.
224. What is the final concentration in the infusion bag (mg/mL)? Round to the nearest tenth decimal.
225. What is the powder volume (mL) of the Funnypenem vial? Round to the nearest tenth decimal place.
Answer: 3.2 mL
To calculate the powder volume of the medication in the vial, subtract the diluent volume (6.8 mL)
from the total reconstituted volume (10 mL).
10mL - 6.8 mL = 3.2 mL